• Matéria: Matemática
  • Autor: Nasgovaskov
  • Perguntado 5 anos atrás

Defina as operações binárias ♣️ e ♡ como

\large\begin{array}{l}\sf a~\clubsuit~b=a^{log_{\:7}\:(b)}\quad e\quad a~\heartsuit~b=a^{\frac{1}{log_{\:7}\:(b)}}\end{array}

para todo número real a e b para os quais essas expressões estão definidas. A sequência (aₙ) é definida recursivamente por

\large\begin{array}{l}\sf a_3=3~\heartsuit~2\quad e\quad a_n=(n~\heartsuit~(n-1))~\clubsuit~a_{n-1}\end{array}

para todo número natural n ≥ 4.

Qual é o número natural mais próximo de log₇ (a₂₀₂₀) ?


Nasgovaskov: obs.: para quem estiver no smartphone, deslize a tela para a esquerda para visualizar a pergunta melhor.
aicitel24: Mano faz um favor para mim apaga minha pergunta por favor

Respostas

respondido por: Anônimo
18

Resposta: o natural mais próximo de log₇ (a₂₀₂₀) é 11 (onze).

O enunciado nos apresenta as operações binárias ♣️ e ♡, definindo-as como

\large\text{$\sf a\ \clubsuit\ b=a^{log_{\,7}\:\!(b)}\quad\, e\quad\ a\ \heartsuit\ b=a^{\frac{1}{log_{\,7}\:\!(b)}}$}

, para todo número real a e b para os quais essas expressões estão definidas. Vimos também que a sequência \boldsymbol{\sf (a_n)} é definida recursivamente por

\large\text{$\sf a_3=3\ \heartsuit\ 2\quad\, e\,\quad a_n=(n\ \heartsuit\ (n-1))\ \clubsuit\ a_{n-1}$}

, para todo natural n ≥ 4. Isto posto, vamos agora calcular o valor de a₃ :

\large\text{$\sf a_3=3\ \heartsuit\ 2$}

\large\text{$\sf a_3=3^{\frac{1}{log_{\,7}\:\!(2)}}$}

\large\text{$\sf a_3=3^{\frac{log_{\,7}\:\!(7)}{log_{\,7}\:\!(2)}}$}

\large\text{$\sf a_3=3^{log_{\,2}\:\!(7)}$}

Seguidamente, temos o valor de a₄ :

\large\text{$\sf a_4=(4\ \heartsuit\ (4-1))\ \clubsuit\ a_{4-1}$}

\large\text{$\sf a_4=(4\ \heartsuit\ 3)\ \clubsuit\ a_3$}

\large\text{$\sf a_4=4^{\frac{1}{log_{\,7}\:\!(3)}}\ \clubsuit\ 3^{log_{\,2}\:\!(7)}$}

\large\text{$\sf a_4=\!\:\!\:\!\left(4^{\frac{1}{log_{\,7}\:\!(3)}}\right)^{\!\!\:\!log_{\,7}\:\!\left(3^{log_{\,2}\:\!(7)}\right)}$}

\large\text{$\sf a_4=\!\:\!\:\!\left(4^{\frac{1}{log_{\,7}\:\!(3)}}\right)^{\!\!\:\!log_{\,2}\:\!(7)\, \cdot\,log_{\,7}\:\!(3)}$}

\large\text{$\sf a_4=4^{\frac{1}{log_{\,7}\:\!(3)}\,\cdot\,log_{\,2}\:\!(7)\,\cdot\,log_{\:\!\:\!7}\:\!(3)}$}

\large\text{$\sf a_4=4^{log_{\,2}\:\!(7)\,\cdot\,\frac{log_{\:\!\:\!7}\:\!(3)}{log_{\:\!\:\!7}\:\!(3)}}$}

\large\text{$\sf a_4=4^{log_{\,2}\:\!(7)\,\cdot\,1}$}

\large\text{$\sf a_4=4^{log_{\,2}\:\!(7)}$}

Com base nos resultados obtidos acima, podemos conjecturar que ∀n ∈ ℕ, com n ≥ 3,

\large\text{$\sf a_n=n^{log_{\,2}\:\!(7)}\qquad \boldsymbol{\sf (\:1\:)}$}

Para provar que ( 1 ) vale para qualquer natural n ≥ 3, recorreremos a seguir ao famigerado Princípio da Indução Finita, comumente designado apenas por PIF. Assim como em toda demonstração via PIF, verificaremos primeiro se a referida fórmula é verdadeira para o elemento mínimo n = 3 (base de indução), depois suporemos sua validade para qualquer natural k ≥ 3 (hipótese de indução) e, por último, tentaremos provar que ela também vale para o sucessor k + 1 de k.

➯ Base de indução

Como vimos, ela vale para n = 3, pois

\large\text{$\sf a_3=3^{log_{\,2}\:\!(7)}\quad\:\, Ok!$}

➯ Hipótese de indução

Admitamos que ( 1 ) seja verdadeira para todo k ≥ 3, ou melhor, ∀k ∈ ℕ, com k ≥ 3, temos

\large\text{$\sf a_k=k^{log_{\,2}\:\!(7)}\qquad\boldsymbol{\sf (\:2\:)}$}

Partindo de \boldsymbol{\sf a_{k+1}} , usando ( 2 ) e a recorrência dada no início desta resolução, encontramos:

\large\text{$\sf a_{k+1}=(k+1\ \heartsuit\ ((k+1)-1))\ \clubsuit\ a_{(k+1)-1}$}

\large\text{$\sf a_{k+1}=(k+1\ \heartsuit\ (k+1-1))\ \clubsuit\ a_{k+1-1}$}

\large\text{$\sf a_{k+1}=(k+1\ \heartsuit\ k)\ \clubsuit\ a_k$}

\large\text{$\sf a_{k+1}=\!\:\!\:\!\left((k+1)^{\frac{1}{log_{\:\!\:\!7}\:\!(k)}}\right)\:\!\:\! \clubsuit\ \,k^{log_{\,2}\:\!(7)}$}

\large\text{$\sf a_{k+1}=\!\:\!\:\!\left((k+1)^{\frac{1}{log_{\:\!\:\!7}\:\!(k)}}\right)^{\!\!\:\!log_{\:\!\:\!7}\left(k^{log_{\,2}\:\!(7)}\right)}$}

\large\text{$\sf a_{k+1}=\!\:\!\:\!\left((k+1)^{\frac{1}{log_{\:\!\:\!7}\:\!(k)}}\right)^{\!\!log_{\,2}\:\!(7)\, \cdot\,log_{\:\!\:\!\:\!7}\:\!(k)}$}

\large\text{$\sf a_{k+1}=(k+1)^{\frac{1}{log_{\,7}\:\!(k)}\,\cdot\,log_{\,2}\:\!(7)\,\cdot\, log_{\:\!\:\!7}\:\!(k)}$}

\large\text{$\sf a_{k+1}=(k+1)^{log_{\:\!\:\!2}\:\!(7)\,\cdot\, \frac{log_{\:\!\:\!7}\:\!(k)}{log_{\:\!\:\!7}\:\!(k)}}$}

\large\text{$\sf a_{k+1}=(k+1)^{log_{\:\!\:\!2}\:\!(7)\,\cdot\,1}$}

\large\text{$\sf a_{k+1}=(k+1)^{log_{\:\!\:\!2}\:\!(7)}\quad\:\,Ok!$}

Ou seja, a fórmula ( 1 ) é válida para k e também para k + 1. Consequentemente, provamos que ∀n ∈ ℕ, com n ≥ 3,

\boldsymbol{\large\text{$\sf a_n=n^{log_{\,2}\:\!(7)}$}}

Sendo assim, log₇ (a₂₀₂₀) será:

\large\text{$\sf log_{\,7}\:\!(a_{2020})=log_{\,7}\!\left(2020^{log_{\,2}\:\!(7)}\right)$}

\large\text{$\sf log_{\,7}\:\!\:\!(a_{2020})\!\:\!=log_{\,2}\:\!(7) \cdot log_{\,7}\:\!(2020)$}

\large\text{$\sf log_{\,7}\:\!\:\!(a_{2020})\!\:\!=log_{\,2}\:\!(7)\cdot \dfrac{log_{\,2}\:\!(2020)}{log_{\,2}\:\!(7)}$}

\large\text{$\sf log_{\,7}\:\!(a_{2020})\!\:\!=log_{\,2}\:\!(2020)$}

Portanto, podemos escrever:

\large\text{$\sf \underbrace{\sf log_{\,2}\:\!(1024)}_{10}<log_{\,2}\:\!(2020)<\underbrace{\sf log_{\,2}\:\!(2048)}_{11}$}

Como vemos, a diferença não negativa entre 2048 e 2020 é 28 e a diferença não negativa entre 2020 e 1024 é 996. Por este motivo, o log₇ (a₂₀₂₀) estará mais próximo do natural log₂ (2048), cujo valor é 11 (onze).

Anexos:

Nasgovaskov: Perfeito amiga !!! Adorei a solução
Anônimo: :)
Anônimo: Tinha um pequeno erro de formatação em uma das equações escritas em LaTeX, mas já foi corrigido. Na hora de calcular o a₄ , eu acabei invertendo o logaritmando com a base, só que isso também já foi corrigido
Perguntas similares